Find k for this probability density function to be valid.

Click For Summary
To determine the constant k for the function f(x) = ke^{-(x-μ)/θ} to be a valid probability density function (pdf) for x > μ, the integral must equal 1. The integration should be performed from μ to infinity, as the function is zero for x < μ. The integral diverges if incorrectly evaluated from -∞ to +∞. After correcting the limits, the solution reveals that k = 1/θ. The discussion concludes with the successful identification of k, ensuring the function meets the pdf criteria.
TelusPig
Messages
15
Reaction score
0

Homework Statement



Find k such that the function f(x)=ke^{-\frac{x-\mu}{\theta}} is a probability density function (pdf), for x &gt; \mu, \mu and \theta are constant.

Homework Equations



The property of a pdf says that the integral of f(x) from -\infty to \infty equals 1, that is \int\limits_{-\infty}^\infty f(x)dx=1

The Attempt at a Solution



\int\limits_{-\infty}^\infty ke^{-\frac{x-\mu}{\theta}}dx
=k\int\limits_{-\infty}^\infty e^{-\frac{x-\mu}{\theta}}dx
Let t=-\frac{x-\mu}{\theta} =&gt; -\theta dt=dx

=&gt; -k\int\limits_\infty^{-\infty} e^t(-\theta)dt

=&gt;k\theta\int\limits_{-\infty}^\infty e^tdt

But e^t would diverge going towards infinity? How could this integral be equal to 1. I am not sure what I'm doing wrong.
 
Physics news on Phys.org
TelusPig said:

Homework Statement



Find k such that the function f(x)=ke^{-\frac{x-\mu}{\theta}} is a probability density function (pdf), for x &gt; \mu, \mu and \theta are constant.

Homework Equations



The property of a pdf says that the integral of f(x) from -\infty to \infty equals 1, that is \int\limits_{-\infty}^\infty f(x)dx=1

The Attempt at a Solution



\int\limits_{-\infty}^\infty ke^{-\frac{x-\mu}{\theta}}dx
=k\int\limits_{-\infty}^\infty e^{-\frac{x-\mu}{\theta}}dx
Let t=-\frac{x-\mu}{\theta} =&gt; -\theta dt=dx

=&gt; -k\int\limits_\infty^{-\infty} e^t(-\theta)dt

=&gt;k\theta\int\limits_{-\infty}^\infty e^tdt

But e^t would diverge going towards infinity? How could this integral be equal to 1. I am not sure what I'm doing wrong.

It's supposed to be a density function only for x>μ. You don't want to integrate from -infinity to +infinity. You want to integrate from μ to +infinity. The pdf will be zero for x<μ.
 
Ok I see what you mean. But even if I do that, the upper bound of infinity is giving me problems because e^(infinity) is infinity :S
 
Never mind! I finally figured it out... I had a -infinity instead. All is good :) I got k = 1/theta if anyone was interested
 
Question: A clock's minute hand has length 4 and its hour hand has length 3. What is the distance between the tips at the moment when it is increasing most rapidly?(Putnam Exam Question) Answer: Making assumption that both the hands moves at constant angular velocities, the answer is ## \sqrt{7} .## But don't you think this assumption is somewhat doubtful and wrong?

Similar threads

  • · Replies 4 ·
Replies
4
Views
1K
  • · Replies 14 ·
Replies
14
Views
2K
Replies
6
Views
2K
  • · Replies 4 ·
Replies
4
Views
2K
Replies
1
Views
2K
Replies
2
Views
2K
  • · Replies 1 ·
Replies
1
Views
2K
  • · Replies 14 ·
Replies
14
Views
3K
  • · Replies 3 ·
Replies
3
Views
1K
  • · Replies 19 ·
Replies
19
Views
4K